GET READY FOR THE AMC 10 WITH AoPS
Learn with outstanding instructors and top-scoring students from around the world in our AMC 10 Problem Series online course.
CHECK SCHEDULE

Difference between revisions of "2003 AMC 10B Problems"

(24 intermediate revisions by 13 users not shown)
Line 1: Line 1:
 +
{{AMC10 Problems|year=2003|ab=B}}
 
==Problem 1==
 
==Problem 1==
  
Line 11: Line 12:
 
==Problem 2==
 
==Problem 2==
  
Al gets the disease algebritis and must take one green pill and one pink pill each day for two weeks. A green pill costs <math>&#36;</math>1<math> more than a pink pill, and Al's pills cost a total of </math>&#36;<math>546</math> for the two weeks. How much does one green pill cost?
+
Al gets the disease algebritis and must take one green pill and one pink pill each day for two weeks. A green pill costs <math> \ </math><math>1</math> more than a pink pill, and Al's pills cost a total of <math> \ </math><math>546</math> for the two weeks. How much does one green pill cost?
  
 
<math> \textbf{(A) }\ </math><math>7 \qquad\textbf{(B) }\ </math> <math>14 \qquad\textbf{(C) }\ </math><math>19\qquad\textbf{(D) }\ </math> <math>20\qquad\textbf{(E) }\ </math><math>39 </math>
 
<math> \textbf{(A) }\ </math><math>7 \qquad\textbf{(B) }\ </math> <math>14 \qquad\textbf{(C) }\ </math><math>19\qquad\textbf{(D) }\ </math> <math>20\qquad\textbf{(E) }\ </math><math>39 </math>
Line 27: Line 28:
 
==Problem 4==
 
==Problem 4==
  
Rose fills each of the rectangular regions of her rectangular flower bed with a different type of flower. The lengths, in feet, of the rectangular regions in her flower bed are as shown in the figure. She plants one flower per square foot in each region. Asters cost <math>&#36;</math>1<math> each, begonias </math>&#36;<math>1.50</math> each, cannas <math>&#36;</math>2<math> each, and Easter lilies </math>&#36;<math>3</math> each. What is the least possible cost, in dollars, for her garden?
+
Rose fills each of the rectangular regions of her rectangular flower bed with a different type of flower. The lengths, in feet, of the rectangular regions in her flower bed are as shown in the figure. She plants one flower per square foot in each region. Asters cost <math> \ </math><math>1</math> each, begonias <math> \ </math><math>1.50</math> each, cannas <math> \ </math><math>2</math> each, dahlias <math> \ </math><math>2.50</math> each, and Easter lilies <math> \ </math><math>3</math> each. What is the least possible cost, in dollars, for her garden?
  
[asy]
+
<asy>
 
unitsize(5mm);
 
unitsize(5mm);
 
defaultpen(linewidth(.8pt)+fontsize(8pt));
 
defaultpen(linewidth(.8pt)+fontsize(8pt));
Line 43: Line 44:
 
label("7",(7.5,6),N);
 
label("7",(7.5,6),N);
 
label("6",(3,0),S);
 
label("6",(3,0),S);
label("5",(8.5,0),S);[/asy]
+
label("5",(8.5,0),S);</asy>
  
 
<math>\textbf{(A) } 108 \qquad\textbf{(B) } 115 \qquad\textbf{(C) } 132 \qquad\textbf{(D) } 144 \qquad\textbf{(E) } 156 </math>
 
<math>\textbf{(A) } 108 \qquad\textbf{(B) } 115 \qquad\textbf{(C) } 132 \qquad\textbf{(D) } 144 \qquad\textbf{(E) } 156 </math>
Line 51: Line 52:
 
==Problem 5==
 
==Problem 5==
  
Moe uses a mower to cut his rectangular <math>90</math>-foot by <math>150</math>-foot lawn. The swath he cuts is <math>28</math> inches wide, but he overlaps each cut by <math>4</math> inches to make sure that no grass is missed. He walks at the rate of <math>5000</math> feet per hour while pushing the mower. Which of the following is closest to the number of hours it will take Moe to mow the lawn.
+
Moe uses a mower to cut his rectangular <math>90</math>-foot by <math>150</math>-foot lawn. The swath he cuts is <math>28</math> inches wide, but he overlaps each cut by <math>4</math> inches to make sure that no grass is missed. He walks at the rate of <math>5000</math> feet per hour while pushing the mower. Which of the following is closest to the number of hours it will take Moe to mow the lawn?
  
 
<math>\textbf{(A) } 0.75 \qquad\textbf{(B) } 0.8 \qquad\textbf{(C) } 1.35 \qquad\textbf{(D) } 1.5 \qquad\textbf{(E) } 3 </math>
 
<math>\textbf{(A) } 0.75 \qquad\textbf{(B) } 0.8 \qquad\textbf{(C) } 1.35 \qquad\textbf{(D) } 1.5 \qquad\textbf{(E) } 3 </math>
Line 61: Line 62:
 
Many television screens are rectangles that are measured by the length of their diagonals. The ratio of the horizontal length to the height in a standard television screen is <math>4 : 3</math>. The horizontal length of a "<math>27</math>-inch" television screen is closest, in inches, to which of the following?
 
Many television screens are rectangles that are measured by the length of their diagonals. The ratio of the horizontal length to the height in a standard television screen is <math>4 : 3</math>. The horizontal length of a "<math>27</math>-inch" television screen is closest, in inches, to which of the following?
  
 +
<asy>
 +
import math;
 +
unitsize(7mm);
 +
defaultpen(linewidth(.8pt)+fontsize(8pt));
 +
draw((0,0)--(4,0)--(4,3)--(0,3)--(0,0)--(4,3));
 +
fill((0,0)--(4,0)--(4,3)--cycle,mediumgray);
 +
label(rotate(aTan(3.0/4.0))*"Diagonal",(2,1.5),NW);
 +
label(rotate(90)*"Height",(4,1.5),E);
 +
label("Length",(2,0),S);</asy>
 
<math>\textbf{(A) } 20 \qquad\textbf{(B) } 20.5 \qquad\textbf{(C) } 21 \qquad\textbf{(D) } 21.5 \qquad\textbf{(E) } 22 </math>
 
<math>\textbf{(A) } 20 \qquad\textbf{(B) } 20.5 \qquad\textbf{(C) } 21 \qquad\textbf{(D) } 21.5 \qquad\textbf{(E) } 22 </math>
  
Line 109: Line 119:
 
==Problem 12==
 
==Problem 12==
  
Al, Betty, and Clare split <math>&#36;</math>1000<math> among them to be invested in different ways. Each begins with a different amount. At the end of one year, they have a total of </math>&#36;<math>1500</math>. Betty and Clare have both doubled their money, whereas Al has managed to lose <math>&#36;</math>100<math>. What was Al's original portion?
+
Al, Betty, and Clare split <math> \ </math><math>1000</math> among them to be invested in different ways. Each begins with a different amount. At the end of one year, they have a total of <math> \ </math><math>1500</math>. Betty and Clare have both doubled their money, whereas Al has managed to lose <math> \ </math><math>100</math>. What was Al's original portion?
  
</math> \textbf{(A) }\ <math></math>250 \qquad\textbf{(B) }\ <math> </math>350 \qquad\textbf{(C) }\ <math></math>400\qquad\textbf{(D) }\ <math> </math>450\qquad\textbf{(E) }\ <math></math>500 <math>
+
<math> \textbf{(A) }\ </math><math>250 \qquad\textbf{(B) }\ </math> <math>350 \qquad\textbf{(C) }\ </math><math>400\qquad\textbf{(D) }\ </math> <math>450\qquad\textbf{(E) }\ </math><math>500 </math>
  
 
[[2003 AMC 10B Problems/Problem 12|Solution]]
 
[[2003 AMC 10B Problems/Problem 12|Solution]]
Line 117: Line 127:
 
==Problem 13==
 
==Problem 13==
  
Let </math>\clubsuit(x)<math> denote the sum of the digits of the positive integer </math>x<math>. For example, </math>\clubsuit(8)=8<math> and </math>\clubsuit(123)=1+2+3=6<math>. For how many two-digit values of </math>x<math> is </math>\clubsuit(\clubsuit(x))=3<math>?
+
Let <math>\clubsuit(x)</math> denote the sum of the digits of the positive integer <math>x</math>. For example, <math>\clubsuit(8)=8</math> and <math>\clubsuit(123)=1+2+3=6</math>. For how many two-digit values of <math>x</math> is <math>\clubsuit(\clubsuit(x))=3</math>?
  
</math>\textbf{(A) } 3 \qquad\textbf{(B) } 4 \qquad\textbf{(C) } 6 \qquad\textbf{(D) } 9 \qquad\textbf{(E) } 10 <math>
+
<math>\textbf{(A) } 3 \qquad\textbf{(B) } 4 \qquad\textbf{(C) } 6 \qquad\textbf{(D) } 9 \qquad\textbf{(E) } 10 </math>
  
 
[[2003 AMC 10B Problems/Problem 13|Solution]]
 
[[2003 AMC 10B Problems/Problem 13|Solution]]
Line 125: Line 135:
 
==Problem 14==
 
==Problem 14==
  
Given that </math>3^8\cdot5^2=a^b,<math> where both </math>a<math> and </math>b<math> are positive integers, find the smallest possible value for </math>a+b<math>.
+
Given that <math>3^8\cdot5^2=a^b,</math> where both <math>a</math> and <math>b</math> are positive integers, find the smallest possible value for <math>a+b</math>.
  
</math>\textbf{(A) } 25 \qquad\textbf{(B) } 34 \qquad\textbf{(C) } 351 \qquad\textbf{(D) } 407 \qquad\textbf{(E) } 900 <math>
+
<math>\textbf{(A) } 25 \qquad\textbf{(B) } 34 \qquad\textbf{(C) } 351 \qquad\textbf{(D) } 407 \qquad\textbf{(E) } 900 </math>
  
 
[[2003 AMC 10B Problems/Problem 14|Solution]]
 
[[2003 AMC 10B Problems/Problem 14|Solution]]
Line 133: Line 143:
 
==Problem 15==
 
==Problem 15==
  
There are </math>100<math> players in a single tennis tournament. The tournament is single elimination, meaning that a player who loses a match is eliminated. In the first round, the strongest </math>28<math> players are given a bye, and the remaining </math>72<math> players are paired off to play. After each round, the remaining players play in the next round. The match continues until only one player remains unbeaten. The total number of matches played is
+
There are <math>100</math> players in a single tennis tournament. The tournament is single elimination, meaning that a player who loses a match is eliminated. In the first round, the strongest <math>28</math> players are given a bye, and the remaining <math>72</math> players are paired off to play. After each round, the remaining players play in the next round. The tournament continues until only one player remains unbeaten. The total number of matches played is
  
</math>\textbf{(A) } \text{a prime number} \qquad\textbf{(B) } \text{divisible by 2} \qquad\textbf{(C) } \text{divisible by 5} \qquad\textbf{(D) } \text{divisible by 7} \qquad\textbf{(E) } \text{divisible by 11}<math>
+
<math>\textbf{(A) } \text{a prime number} \qquad\textbf{(B) } \text{divisible by 2} \qquad\textbf{(C) } \text{divisible by 5} \qquad\textbf{(D) } \text{divisible by 7} \qquad\textbf{(E) } \text{divisible by 11}</math>
  
 
[[2003 AMC 10B Problems/Problem 15|Solution]]
 
[[2003 AMC 10B Problems/Problem 15|Solution]]
Line 141: Line 151:
 
==Problem 16==
 
==Problem 16==
  
A restaurant offers three deserts, and exactly twice as many appetizers as main courses. A dinner consists of an appetizer, a main course, and a dessert. What is the least number of main courses that a restaurant should offer so that a customer could have a different dinner each night in the year </math>2003<math>?
+
A restaurant offers three desserts, and exactly twice as many appetizers as main courses. A dinner consists of an appetizer, a main course, and a dessert. What is the least number of main courses that a restaurant should offer so that a customer could have a different dinner each night in the year <math>2003</math>?
  
</math>\textbf{(A) } 4 \qquad\textbf{(B) } 5 \qquad\textbf{(C) } 6 \qquad\textbf{(D) } 7 \qquad\textbf{(E) } 8<math>
+
<math>\textbf{(A) } 4 \qquad\textbf{(B) } 5 \qquad\textbf{(C) } 6 \qquad\textbf{(D) } 7 \qquad\textbf{(E) } 8</math>
  
 
[[2003 AMC 10B Problems/Problem 16|Solution]]
 
[[2003 AMC 10B Problems/Problem 16|Solution]]
Line 149: Line 159:
 
==Problem 17==
 
==Problem 17==
  
An ice cream cone consists of a sphere of vanilla ice cream and a right circular cone that has the same diameter as the sphere. If the ice cream melts, it will exactly fill the cone. Assume that the melted ice cream occupies </math>75\%<math> of the volume of the frozen ice cream. What is the ratio of the cone's height to its radius? (Note: a cone with radius </math>r<math> and height </math>h<math> has volume </math>\pi r^2 h / 3<math> and a sphere with radius </math>r<math> has volume </math>4 \pi r^3 / 3<math>.)
+
An ice cream cone consists of a sphere of vanilla ice cream and a right circular cone that has the same diameter as the sphere. If the ice cream melts, it will exactly fill the cone. Assume that the melted ice cream occupies <math>75\%</math> of the volume of the frozen ice cream. What is the ratio of the cone's height to its radius? (Note: a cone with radius <math>r</math> and height <math>h</math> has volume <math>\pi r^2 h / 3</math> and a sphere with radius <math>r</math> has volume <math>4 \pi r^3 / 3</math>.)
  
</math>\textbf{(A) } 2:1 \qquad\textbf{(B) } 3:1 \qquad\textbf{(C) } 4:1 \qquad\textbf{(D) } 16:3 \qquad\textbf{(E) } 6:1 <math>
+
<math>\textbf{(A) } 2:1 \qquad\textbf{(B) } 3:1 \qquad\textbf{(C) } 4:1 \qquad\textbf{(D) } 16:3 \qquad\textbf{(E) } 6:1 </math>
  
 
[[2003 AMC 10B Problems/Problem 17|Solution]]
 
[[2003 AMC 10B Problems/Problem 17|Solution]]
Line 159: Line 169:
 
What is the largest integer that is a divisor of
 
What is the largest integer that is a divisor of
 
<cmath> (n+1)(n+3)(n+5)(n+7)(n+9) </cmath>
 
<cmath> (n+1)(n+3)(n+5)(n+7)(n+9) </cmath>
for all positive even integers </math>n<math>?
+
for all positive even integers <math>n</math>?
  
</math>\textbf{(A) } 3 \qquad\textbf{(B) } 5 \qquad\textbf{(C) } 11 \qquad\textbf{(D) } 15 \qquad\textbf{(E) } 165<math>
+
<math>\textbf{(A) } 3 \qquad\textbf{(B) } 5 \qquad\textbf{(C) } 11 \qquad\textbf{(D) } 15 \qquad\textbf{(E) } 165</math>
  
 
[[2003 AMC 10B Problems/Problem 18|Solution]]
 
[[2003 AMC 10B Problems/Problem 18|Solution]]
Line 167: Line 177:
 
==Problem 19==
 
==Problem 19==
  
Three semicircles of radius </math>1<math> are constructed on diameter </math>\overline{AB}<math> of a semicircle of radius </math>2<math>. The centers of the small semicircles divide </math>\overline{AB}<math> into four line segments of equal length, as shown. What is the area of the shaded region that lies within the large semicircle but outside the smaller semicircles?
+
Three semicircles of radius <math>1</math> are constructed on diameter <math>\overline{AB}</math> of a semicircle of radius <math>2</math>. The centers of the small semicircles divide <math>\overline{AB}</math> into four line segments of equal length, as shown. What is the area of the shaded region that lies within the large semicircle but outside the smaller semicircles?
 +
 
 +
<asy>
 +
import graph;
 +
unitsize(14mm);
 +
defaultpen(linewidth(.8pt)+fontsize(8pt));
 +
dashed=linetype("4 4");
 +
dotfactor=3;
 +
pair A=(-2,0), B=(2,0);
 +
fill(Arc((0,0),2,0,180)--cycle,mediumgray);
 +
fill(Arc((-1,0),1,0,180)--cycle,white);
 +
fill(Arc((0,0),1,0,180)--cycle,white);
 +
fill(Arc((1,0),1,0,180)--cycle,white);
 +
draw(Arc((-1,0),1,60,180));
 +
draw(Arc((0,0),1,0,60),dashed);
 +
draw(Arc((0,0),1,60,120));
 +
draw(Arc((0,0),1,120,180),dashed);
 +
draw(Arc((1,0),1,0,120));
 +
draw(Arc((0,0),2,0,180)--cycle);
 +
dot((0,0));
 +
dot((-1,0));
 +
dot((1,0));
 +
draw((-2,-0.1)--(-2,-0.3),gray);
 +
draw((-1,-0.1)--(-1,-0.3),gray);
 +
draw((1,-0.1)--(1,-0.3),gray);
 +
draw((2,-0.1)--(2,-0.3),gray);
 +
label("$A$",A,W);
 +
label("$B$",B,E);
 +
label("1",(-1.5,-0.1),S);
 +
label("2",(0,-0.1),S);
 +
label("1",(1.5,-0.1),S);</asy>
  
</math>\textbf{(A) } \pi - \sqrt{3} \qquad\textbf{(B) } \pi - \sqrt{2} \qquad\textbf{(C) } \frac{\pi + \sqrt{2}}{2} \qquad\textbf{(D) } \frac{\pi +\sqrt{3}}{2} \qquad\textbf{(E) } \frac{7}{6}\pi - \frac{\sqrt{3}}{2}<math>
+
<math>\textbf{(A) } \pi - \sqrt{3} \qquad\textbf{(B) } \pi - \sqrt{2} \qquad\textbf{(C) } \frac{\pi + \sqrt{2}}{2} \qquad\textbf{(D) } \frac{\pi +\sqrt{3}}{2} \qquad\textbf{(E) } \frac{7}{6}\pi - \frac{\sqrt{3}}{2}</math>
  
 
[[2003 AMC 10B Problems/Problem 19|Solution]]
 
[[2003 AMC 10B Problems/Problem 19|Solution]]
Line 175: Line 215:
 
==Problem 20==
 
==Problem 20==
  
In rectangle </math>ABCD, AB=5<math> and </math>BC=3<math>. Points </math>F<math> and </math>G<math> are on </math>\overline{CD}<math> so that </math>DF=1<math> and </math>GC=2<math>. Lines </math>AF<math> and </math>BG<math> intersect at </math>E<math>. Find the area of </math>\triangle AEB<math>.
+
In rectangle <math>ABCD, AB=5</math> and <math>BC=3</math>. Points <math>F</math> and <math>G</math> are on <math>\overline{CD}</math> so that <math>DF=1</math> and <math>GC=2</math>. Lines <math>AF</math> and <math>BG</math> intersect at <math>E</math>. Find the area of <math>\triangle AEB</math>.
  
</math>\textbf{(A) } 10 \qquad\textbf{(B) } \frac{21}{2} \qquad\textbf{(C) } 12 \qquad\textbf{(D) } \frac{25}{2} \qquad\textbf{(E) } 15<math>
+
<asy>
 +
unitsize(6mm);
 +
defaultpen(linewidth(.8pt)+fontsize(8pt));
 +
pair A=(0,0), B=(5,0), C=(5,3), D=(0,3), F=(1,3), G=(3,3);
 +
pair E=extension(A,F,B,G);
 +
draw(A--B--C--D--A--E--B);
 +
label("$A$",A,SW);
 +
label("$B$",B,SE);
 +
label("$C$",C,NE);
 +
label("$D$",D,NW);
 +
label("$E$",E,N);
 +
label("$F$",F,SE);
 +
label("$G$",G,SW);
 +
label("$B$",B,SE);
 +
label("1",midpoint(D--F),N);
 +
label("2",midpoint(G--C),N);
 +
label("3",midpoint(B--C),E);
 +
label("3",midpoint(A--D),W);
 +
label("5",midpoint(A--B),S); </asy>
 +
<math>\textbf{(A) } 10 \qquad\textbf{(B) } \frac{21}{2} \qquad\textbf{(C) } 12 \qquad\textbf{(D) } \frac{25}{2} \qquad\textbf{(E) } 15</math>
  
 
[[2003 AMC 10B Problems/Problem 20|Solution]]
 
[[2003 AMC 10B Problems/Problem 20|Solution]]
Line 185: Line 244:
 
A bag contains two red beads and two green beads. You reach into the bag and pull out a bead, replacing it with a red bead regardless of the color you pulled out. What is the probability that all beads in the bag are red after three such replacements?
 
A bag contains two red beads and two green beads. You reach into the bag and pull out a bead, replacing it with a red bead regardless of the color you pulled out. What is the probability that all beads in the bag are red after three such replacements?
  
</math>\textbf{(A) } \frac{1}{8} \qquad\textbf{(B) } \frac{5}{32} \qquad\textbf{(C) } \frac{9}{32} \qquad\textbf{(D) } \frac{3}{8} \qquad\textbf{(E) } \frac{7}{16}<math>
+
<math>\textbf{(A) } \frac{1}{8} \qquad\textbf{(B) } \frac{5}{32} \qquad\textbf{(C) } \frac{9}{32} \qquad\textbf{(D) } \frac{3}{8} \qquad\textbf{(E) } \frac{7}{16}</math>
  
 
[[2003 AMC 10B Problems/Problem 21|Solution]]
 
[[2003 AMC 10B Problems/Problem 21|Solution]]
Line 191: Line 250:
 
==Problem 22==
 
==Problem 22==
  
A clock chimes once at </math>30<math> minutes past each hour and chimes on the hour according to the hour. For example, at </math>1 \text{PM}<math> there is one chime and at noon and midnight there are twelve chimes. Starting at </math>11:15 \text{AM}<math> on </math>\text{February 26, 2003},<math> on what date will the </math>2003\text{rd}<math> chime occur?
+
A clock chimes once at <math>30</math> minutes past each hour and chimes on the hour according to the hour. For example, at <math>1 \text{PM}</math> there is one chime and at noon and midnight there are twelve chimes. Starting at <math>11:15 \text{AM}</math> on <math>\text{February 26, 2003},</math> on what date will the <math>2003\text{rd}</math> chime occur?
  
</math>\textbf{(A) } \text{March 8} \qquad\textbf{(B) } \text{March 9} \qquad\textbf{(C) } \text{March 10} \qquad\textbf{(D) } \text{March 20} \qquad\textbf{(E) } \text{March 21}<math>
+
<math>\textbf{(A) } \text{March 8} \qquad\textbf{(B) } \text{March 9} \qquad\textbf{(C) } \text{March 10} \qquad\textbf{(D) } \text{March 20} \qquad\textbf{(E) } \text{March 21}</math>
  
 
[[2003 AMC 10B Problems/Problem 22|Solution]]
 
[[2003 AMC 10B Problems/Problem 22|Solution]]
Line 199: Line 258:
 
==Problem 23==
 
==Problem 23==
  
A regular octagon </math>ABCDEFGH<math> has an area of one square unit. What is the area of the rectangle </math>ABEF<math>?
+
A regular octagon <math>ABCDEFGH</math> has an area of one square unit. What is the area of the rectangle <math>ABEF</math>?
  
</math>\textbf{(A) } 1 - \frac{\sqrt{2}}{2} \qquad\textbf{(B) } \frac{\sqrt{2}}{4} \qquad\textbf{(C) } \sqrt{2} - 1 \qquad\textbf{(D) } \frac{1}{2} \qquad\textbf{(E) } \frac{1+\sqrt{2}}{4}<math>
+
<asy>
 +
unitsize(10mm);
 +
defaultpen(linewidth(.8pt)+fontsize(8pt));
 +
pair C=dir(22.5), B=dir(67.5), A=dir(112.5), H=dir(157.5), G=dir(202.5), F=dir(247.5), E=dir(292.5), D=dir(337.5);
 +
draw(A--B--C--D--E--F--G--H--cycle);
 +
label("$A$",A,NNW);
 +
label("$B$",B,NNE);
 +
label("$C$",C,ENE);
 +
label("$D$",D,ESE);
 +
label("$E$",E,SSE);
 +
label("$F$",F,SSW);
 +
label("$G$",G,WSW);
 +
label("$H$",H,WNW); </asy>
 +
<math>\textbf{(A) } 1 - \frac{\sqrt{2}}{2} \qquad\textbf{(B) } \frac{\sqrt{2}}{4} \qquad\textbf{(C) } \sqrt{2} - 1 \qquad\textbf{(D) } \frac{1}{2} \qquad\textbf{(E) } \frac{1+\sqrt{2}}{4}</math>
  
 
[[2003 AMC 10B Problems/Problem 23|Solution]]
 
[[2003 AMC 10B Problems/Problem 23|Solution]]
Line 207: Line 279:
 
==Problem 24==
 
==Problem 24==
  
The first four terms in an arithmetic sequence are </math>x + y, x - y, xy,<math> and </math>x/y,<math> in that order. What is the fifth term?
+
The first four terms in an arithmetic sequence are <math>x + y, x - y, xy,</math> and <math>x/y,</math> in that order. What is the fifth term?  
  
</math>\textbf{(A) } -\frac{15}{8} \qquad\textbf{(B) } -\frac{6}{5} \qquad\textbf{(C) } 0 \qquad\textbf{(D) } \frac{27}{20} \qquad\textbf{(E) } \frac{123}{40}<math>
+
<math> \textbf{(A)}\ -\frac{15}{8}\qquad\textbf{(B)}\ -\frac{6}{5}\qquad\textbf{(C)}\ 0\qquad\textbf{(D)}\ \frac{27}{20}\qquad\textbf{(E)}\ \frac{123}{40} </math>
  
 
[[2003 AMC 10B Problems/Problem 24|Solution]]
 
[[2003 AMC 10B Problems/Problem 24|Solution]]
Line 215: Line 287:
 
==Problem 25==
 
==Problem 25==
  
How many distinct four-digit numbers are divisible by </math>3<math> and have </math>23<math> as their last two digits?
+
How many distinct four-digit numbers are divisible by <math>3</math> and have <math>23</math> as their last two digits?
  
</math>\textbf{(A) } 27 \qquad\textbf{(B) } 30 \qquad\textbf{(C) } 33 \qquad\textbf{(D) } 81 \qquad\textbf{(E) } 90$
+
<math>\textbf{(A) } 27 \qquad\textbf{(B) } 30 \qquad\textbf{(C) } 33 \qquad\textbf{(D) } 81 \qquad\textbf{(E) } 90</math>
  
 
[[2003 AMC 10B Problems/Problem 25|Solution]]
 
[[2003 AMC 10B Problems/Problem 25|Solution]]
 +
 +
== See also ==
 +
{{AMC10 box|year=2003|ab=B|before=[[2003 AMC 10A Problems]]|after=[[2004 AMC 10A Problems]]}}
 +
* [[AMC 10]]
 +
* [[AMC 10 Problems and Solutions]]
 +
* [[AMC Problems and Solutions]]
 +
* [[Mathematics competition resources]]
 +
 +
{{MAA Notice}}

Revision as of 13:05, 19 February 2020

2003 AMC 10B (Answer Key)
Printable versions: WikiAoPS ResourcesPDF

Instructions

  1. This is a 25-question, multiple choice test. Each question is followed by answers marked A, B, C, D and E. Only one of these is correct.
  2. You will receive 6 points for each correct answer, 2.5 points for each problem left unanswered if the year is before 2006, 1.5 points for each problem left unanswered if the year is after 2006, and 0 points for each incorrect answer.
  3. No aids are permitted other than scratch paper, graph paper, ruler, compass, protractor and erasers (and calculators that are accepted for use on the SAT if before 2006. No problems on the test will require the use of a calculator).
  4. Figures are not necessarily drawn to scale.
  5. You will have 75 minutes working time to complete the test.
1 2 3 4 5 6 7 8 9 10 11 12 13 14 15 16 17 18 19 20 21 22 23 24 25

Problem 1

Which of the following is the same as

\[\frac{2-4+6-8+10-12+14}{3-6+9-12+15-18+21}?\]

$\textbf{(A)} -1 \qquad\textbf{(B)} -\frac{2}{3} \qquad\textbf{(C)} \frac{2}{3} \qquad\textbf{(D)} 1 \qquad\textbf{(E)} \frac{14}{3}$

Solution

Problem 2

Al gets the disease algebritis and must take one green pill and one pink pill each day for two weeks. A green pill costs $$$1$ more than a pink pill, and Al's pills cost a total of $$$546$ for the two weeks. How much does one green pill cost?

$\textbf{(A) }$$7 \qquad\textbf{(B) }$ $14 \qquad\textbf{(C) }$$19\qquad\textbf{(D) }$ $20\qquad\textbf{(E) }$$39$

Solution

Problem 3

The sum of $5$ consecutive even integers is $4$ less than the sum of the first $8$ consecutive odd counting numbers. What is the smallest of the even integers?

$\textbf{(A) } 6 \qquad\textbf{(B) } 8 \qquad\textbf{(C) } 10 \qquad\textbf{(D) } 12 \qquad\textbf{(E) } 14$

Solution

Problem 4

Rose fills each of the rectangular regions of her rectangular flower bed with a different type of flower. The lengths, in feet, of the rectangular regions in her flower bed are as shown in the figure. She plants one flower per square foot in each region. Asters cost $$$1$ each, begonias $$$1.50$ each, cannas $$$2$ each, dahlias $$$2.50$ each, and Easter lilies $$$3$ each. What is the least possible cost, in dollars, for her garden?

[asy] unitsize(5mm); defaultpen(linewidth(.8pt)+fontsize(8pt)); draw((6,0)--(0,0)--(0,1)--(6,1)); draw((0,1)--(0,6)--(4,6)--(4,1)); draw((4,6)--(11,6)--(11,3)--(4,3)); draw((11,3)--(11,0)--(6,0)--(6,3)); label("1",(0,0.5),W); label("5",(0,3.5),W); label("3",(11,1.5),E); label("3",(11,4.5),E); label("4",(2,6),N); label("7",(7.5,6),N); label("6",(3,0),S); label("5",(8.5,0),S);[/asy]

$\textbf{(A) } 108 \qquad\textbf{(B) } 115 \qquad\textbf{(C) } 132 \qquad\textbf{(D) } 144 \qquad\textbf{(E) } 156$

Solution

Problem 5

Moe uses a mower to cut his rectangular $90$-foot by $150$-foot lawn. The swath he cuts is $28$ inches wide, but he overlaps each cut by $4$ inches to make sure that no grass is missed. He walks at the rate of $5000$ feet per hour while pushing the mower. Which of the following is closest to the number of hours it will take Moe to mow the lawn?

$\textbf{(A) } 0.75 \qquad\textbf{(B) } 0.8 \qquad\textbf{(C) } 1.35 \qquad\textbf{(D) } 1.5 \qquad\textbf{(E) } 3$

Solution

Problem 6

Many television screens are rectangles that are measured by the length of their diagonals. The ratio of the horizontal length to the height in a standard television screen is $4 : 3$. The horizontal length of a "$27$-inch" television screen is closest, in inches, to which of the following?

[asy]  import math; unitsize(7mm); defaultpen(linewidth(.8pt)+fontsize(8pt)); draw((0,0)--(4,0)--(4,3)--(0,3)--(0,0)--(4,3)); fill((0,0)--(4,0)--(4,3)--cycle,mediumgray); label(rotate(aTan(3.0/4.0))*"Diagonal",(2,1.5),NW); label(rotate(90)*"Height",(4,1.5),E); label("Length",(2,0),S);[/asy] $\textbf{(A) } 20 \qquad\textbf{(B) } 20.5 \qquad\textbf{(C) } 21 \qquad\textbf{(D) } 21.5 \qquad\textbf{(E) } 22$

Solution

Problem 7

The symbolism $\lfloor x \rfloor$ denotes the largest integer not exceeding $x$. For example, $\lfloor 3 \rfloor = 3,$ and $\lfloor 9/2 \rfloor = 4$. Compute \[\lfloor \sqrt{1} \rfloor + \lfloor \sqrt{2} \rfloor + \lfloor \sqrt{3} \rfloor + \cdots + \lfloor \sqrt{16} \rfloor.\]

$\textbf{(A) } 35 \qquad\textbf{(B) } 38 \qquad\textbf{(C) } 40 \qquad\textbf{(D) } 42 \qquad\textbf{(E) } 136$

Solution

Problem 8

The second and fourth terms of a geometric sequence are $2$ and $6$. Which of the following is a possible first term?

$\textbf{(A) } -\sqrt{3}  \qquad\textbf{(B) } -\frac{2\sqrt{3}}{3} \qquad\textbf{(C) } -\frac{\sqrt{3}}{3} \qquad\textbf{(D) } \sqrt{3} \qquad\textbf{(E) } 3$

Solution

Problem 9

Find the value of $x$ that satisfies the equation \[25^{-2} = \frac{5^{48/x}}{5^{26/x} \cdot 25^{17/x}}.\]

$\textbf{(A) } 2 \qquad\textbf{(B) } 3 \qquad\textbf{(C) } 5 \qquad\textbf{(D) } 6 \qquad\textbf{(E) } 9$

Solution

Problem 10

Nebraska, the home of the AMC, changed its license plate scheme. Each old license plate consisted of a letter followed by four digits. Each new license plate consists of the three letters followed by three digits. By how many times is the number of possible license plates increased?

$\textbf{(A) } \frac{26}{10} \qquad\textbf{(B) } \frac{26^2}{10^2} \qquad\textbf{(C) } \frac{26^2}{10} \qquad\textbf{(D) } \frac{26^3}{10^3} \qquad\textbf{(E) } \frac{26^3}{10^2}$

Solution

Problem 11

A line with slope $3$ intersects a line with slope $5$ at point $(10,15)$. What is the distance between the $x$-intercepts of these two lines?

$\textbf{(A) } 2 \qquad\textbf{(B) } 5 \qquad\textbf{(C) } 7 \qquad\textbf{(D) } 12 \qquad\textbf{(E) } 20$

Solution

Problem 12

Al, Betty, and Clare split $$$1000$ among them to be invested in different ways. Each begins with a different amount. At the end of one year, they have a total of $$$1500$. Betty and Clare have both doubled their money, whereas Al has managed to lose $$$100$. What was Al's original portion?

$\textbf{(A) }$$250 \qquad\textbf{(B) }$ $350 \qquad\textbf{(C) }$$400\qquad\textbf{(D) }$ $450\qquad\textbf{(E) }$$500$

Solution

Problem 13

Let $\clubsuit(x)$ denote the sum of the digits of the positive integer $x$. For example, $\clubsuit(8)=8$ and $\clubsuit(123)=1+2+3=6$. For how many two-digit values of $x$ is $\clubsuit(\clubsuit(x))=3$?

$\textbf{(A) } 3 \qquad\textbf{(B) } 4 \qquad\textbf{(C) } 6 \qquad\textbf{(D) } 9 \qquad\textbf{(E) } 10$

Solution

Problem 14

Given that $3^8\cdot5^2=a^b,$ where both $a$ and $b$ are positive integers, find the smallest possible value for $a+b$.

$\textbf{(A) } 25 \qquad\textbf{(B) } 34 \qquad\textbf{(C) } 351 \qquad\textbf{(D) } 407 \qquad\textbf{(E) } 900$

Solution

Problem 15

There are $100$ players in a single tennis tournament. The tournament is single elimination, meaning that a player who loses a match is eliminated. In the first round, the strongest $28$ players are given a bye, and the remaining $72$ players are paired off to play. After each round, the remaining players play in the next round. The tournament continues until only one player remains unbeaten. The total number of matches played is

$\textbf{(A) } \text{a prime number} \qquad\textbf{(B) } \text{divisible by 2} \qquad\textbf{(C) } \text{divisible by 5} \qquad\textbf{(D) } \text{divisible by 7} \qquad\textbf{(E) } \text{divisible by 11}$

Solution

Problem 16

A restaurant offers three desserts, and exactly twice as many appetizers as main courses. A dinner consists of an appetizer, a main course, and a dessert. What is the least number of main courses that a restaurant should offer so that a customer could have a different dinner each night in the year $2003$?

$\textbf{(A) } 4 \qquad\textbf{(B) } 5 \qquad\textbf{(C) } 6 \qquad\textbf{(D) } 7 \qquad\textbf{(E) } 8$

Solution

Problem 17

An ice cream cone consists of a sphere of vanilla ice cream and a right circular cone that has the same diameter as the sphere. If the ice cream melts, it will exactly fill the cone. Assume that the melted ice cream occupies $75\%$ of the volume of the frozen ice cream. What is the ratio of the cone's height to its radius? (Note: a cone with radius $r$ and height $h$ has volume $\pi r^2 h / 3$ and a sphere with radius $r$ has volume $4 \pi r^3 / 3$.)

$\textbf{(A) } 2:1 \qquad\textbf{(B) } 3:1 \qquad\textbf{(C) } 4:1 \qquad\textbf{(D) } 16:3 \qquad\textbf{(E) } 6:1$

Solution

Problem 18

What is the largest integer that is a divisor of \[(n+1)(n+3)(n+5)(n+7)(n+9)\] for all positive even integers $n$?

$\textbf{(A) } 3 \qquad\textbf{(B) } 5 \qquad\textbf{(C) } 11 \qquad\textbf{(D) } 15 \qquad\textbf{(E) } 165$

Solution

Problem 19

Three semicircles of radius $1$ are constructed on diameter $\overline{AB}$ of a semicircle of radius $2$. The centers of the small semicircles divide $\overline{AB}$ into four line segments of equal length, as shown. What is the area of the shaded region that lies within the large semicircle but outside the smaller semicircles?

[asy] import graph; unitsize(14mm); defaultpen(linewidth(.8pt)+fontsize(8pt)); dashed=linetype("4 4"); dotfactor=3; pair A=(-2,0), B=(2,0); fill(Arc((0,0),2,0,180)--cycle,mediumgray); fill(Arc((-1,0),1,0,180)--cycle,white); fill(Arc((0,0),1,0,180)--cycle,white); fill(Arc((1,0),1,0,180)--cycle,white); draw(Arc((-1,0),1,60,180)); draw(Arc((0,0),1,0,60),dashed); draw(Arc((0,0),1,60,120)); draw(Arc((0,0),1,120,180),dashed); draw(Arc((1,0),1,0,120)); draw(Arc((0,0),2,0,180)--cycle); dot((0,0)); dot((-1,0)); dot((1,0)); draw((-2,-0.1)--(-2,-0.3),gray); draw((-1,-0.1)--(-1,-0.3),gray); draw((1,-0.1)--(1,-0.3),gray); draw((2,-0.1)--(2,-0.3),gray); label("$A$",A,W); label("$B$",B,E); label("1",(-1.5,-0.1),S); label("2",(0,-0.1),S); label("1",(1.5,-0.1),S);[/asy]

$\textbf{(A) } \pi - \sqrt{3} \qquad\textbf{(B) } \pi - \sqrt{2} \qquad\textbf{(C) } \frac{\pi + \sqrt{2}}{2} \qquad\textbf{(D) } \frac{\pi +\sqrt{3}}{2} \qquad\textbf{(E) } \frac{7}{6}\pi - \frac{\sqrt{3}}{2}$

Solution

Problem 20

In rectangle $ABCD, AB=5$ and $BC=3$. Points $F$ and $G$ are on $\overline{CD}$ so that $DF=1$ and $GC=2$. Lines $AF$ and $BG$ intersect at $E$. Find the area of $\triangle AEB$.

[asy]  unitsize(6mm); defaultpen(linewidth(.8pt)+fontsize(8pt)); pair A=(0,0), B=(5,0), C=(5,3), D=(0,3), F=(1,3), G=(3,3); pair E=extension(A,F,B,G); draw(A--B--C--D--A--E--B); label("$A$",A,SW); label("$B$",B,SE); label("$C$",C,NE); label("$D$",D,NW); label("$E$",E,N); label("$F$",F,SE); label("$G$",G,SW); label("$B$",B,SE); label("1",midpoint(D--F),N); label("2",midpoint(G--C),N); label("3",midpoint(B--C),E); label("3",midpoint(A--D),W); label("5",midpoint(A--B),S); [/asy] $\textbf{(A) } 10 \qquad\textbf{(B) } \frac{21}{2} \qquad\textbf{(C) } 12 \qquad\textbf{(D) } \frac{25}{2} \qquad\textbf{(E) } 15$

Solution

Problem 21

A bag contains two red beads and two green beads. You reach into the bag and pull out a bead, replacing it with a red bead regardless of the color you pulled out. What is the probability that all beads in the bag are red after three such replacements?

$\textbf{(A) } \frac{1}{8} \qquad\textbf{(B) } \frac{5}{32} \qquad\textbf{(C) } \frac{9}{32} \qquad\textbf{(D) } \frac{3}{8} \qquad\textbf{(E) } \frac{7}{16}$

Solution

Problem 22

A clock chimes once at $30$ minutes past each hour and chimes on the hour according to the hour. For example, at $1 \text{PM}$ there is one chime and at noon and midnight there are twelve chimes. Starting at $11:15 \text{AM}$ on $\text{February 26, 2003},$ on what date will the $2003\text{rd}$ chime occur?

$\textbf{(A) } \text{March 8} \qquad\textbf{(B) } \text{March 9} \qquad\textbf{(C) } \text{March 10} \qquad\textbf{(D) } \text{March 20} \qquad\textbf{(E) } \text{March 21}$

Solution

Problem 23

A regular octagon $ABCDEFGH$ has an area of one square unit. What is the area of the rectangle $ABEF$?

[asy] unitsize(10mm); defaultpen(linewidth(.8pt)+fontsize(8pt)); pair C=dir(22.5), B=dir(67.5), A=dir(112.5), H=dir(157.5), G=dir(202.5), F=dir(247.5), E=dir(292.5), D=dir(337.5); draw(A--B--C--D--E--F--G--H--cycle); label("$A$",A,NNW); label("$B$",B,NNE); label("$C$",C,ENE); label("$D$",D,ESE); label("$E$",E,SSE); label("$F$",F,SSW); label("$G$",G,WSW); label("$H$",H,WNW); [/asy] $\textbf{(A) } 1 - \frac{\sqrt{2}}{2} \qquad\textbf{(B) } \frac{\sqrt{2}}{4} \qquad\textbf{(C) } \sqrt{2} - 1 \qquad\textbf{(D) } \frac{1}{2} \qquad\textbf{(E) } \frac{1+\sqrt{2}}{4}$

Solution

Problem 24

The first four terms in an arithmetic sequence are $x + y, x - y, xy,$ and $x/y,$ in that order. What is the fifth term?

$\textbf{(A)}\ -\frac{15}{8}\qquad\textbf{(B)}\ -\frac{6}{5}\qquad\textbf{(C)}\ 0\qquad\textbf{(D)}\ \frac{27}{20}\qquad\textbf{(E)}\ \frac{123}{40}$

Solution

Problem 25

How many distinct four-digit numbers are divisible by $3$ and have $23$ as their last two digits?

$\textbf{(A) } 27 \qquad\textbf{(B) } 30 \qquad\textbf{(C) } 33 \qquad\textbf{(D) } 81 \qquad\textbf{(E) } 90$

Solution

See also

2003 AMC 10B (ProblemsAnswer KeyResources)
Preceded by
2003 AMC 10A Problems
Followed by
2004 AMC 10A Problems
1 2 3 4 5 6 7 8 9 10 11 12 13 14 15 16 17 18 19 20 21 22 23 24 25
All AMC 10 Problems and Solutions

The problems on this page are copyrighted by the Mathematical Association of America's American Mathematics Competitions. AMC logo.png